In Exercises 19-22, two triangles can be formed using the given meas- urements. Solve both triangles. 14. 19. A = 64°, a = 16,. 20. B 38°,. 21. C 68°,.

Answers

Answer 1

Two triangles can be formed using the given measurements,

19. Triangle 1: A = 64°, B ≈ 53.07°, C ≈ 62.93°, a = 16, b ≈ 14.83, c ≈ 16.64

Triangle 2: A = 64°, B ≈ 126.93°, C ≈ 9.07°, a = 16, b ≈ 80.17, c ≈ 8.98

20. Triangle 1: A ≈ 52°, B = 38°, C ≈ 94°, a ≈ 22.57, b = b, c ≈ 34.60

Triangle 2: A ≈ 128°, B = 38°, C ≈ 14°, a ≈ 22.57, b = b, c ≈ 16.66

19. We are given angle A and the side opposite to it, a. We can use the law of sines to find the other sides and angles of the triangle:

a/sin(A) = b/sin(B) = c/sin(C)

b/sin(B) = a/sin(A)

b = a × sin(B)/sin(A)

b = 16 × sin(64°)/sin(180°-64°-90°)

b ≈ 14.83

c/sin(C) = a/sin(A)

c = a × sin(C)/sin(A)

c = 16 × sin(68°)/sin(64°)

c ≈ 16.64

Therefore, the two triangles are:

Triangle 1: A = 64°, B ≈ 53.07°, C ≈ 62.93°, a = 16, b ≈ 14.83, c ≈ 16.64

Triangle 2: A = 64°, B ≈ 126.93°, C ≈ 9.07°, a = 16, b ≈ 80.17, c ≈ 8.98

20. We are given angle B. Let the length of the side opposite to B be b. We can use the fact that the angles in a triangle add up to 180° to find angle A, and then use the law of sines to find the remaining sides and angles:

A = 180° - 90° - 38°

A = 52°

a/sin(A) = b/sin(B) = c/sin(C)

a/sin(52°) = b/sin(38°)

c/sin(C) = b/sin(38°)

c = b*sin(C)/sin(38°)

The angles in a triangle add up to 180°, so we have:

C = 180° - A - B

C ≈ 94°

Substituting the values of A, B, and C in the above equations, we get:

a ≈ 22.57

b = b

c ≈ 34.60

Therefore, the two triangles are:

Triangle 1: A ≈ 52°, B = 38°, C ≈ 94°, a ≈ 22.57, b = b, c ≈ 34.60

Triangle 2: A ≈ 128°, B = 38°, C ≈ 14°, a ≈ 22.57, b = b, c ≈ 16.66

Learn more about triangles at

https://brainly.com/question/2773823

#SPJ4

The question is -

In Exercises 19-22, two triangles can be formed using the given measurements.

Solve both triangles.

19. A = 64°, a = 16,

20. B 38°


Related Questions

Answer this question pls due soon

Answers

The height of the monitor using Pythagoras theorem is: 13.3 inches

How to use Pythagoras theorem?

Pythagoras Theorem is defined as the way in which you can find the missing length of a right angled triangle.

The triangle has three sides, the hypotenuse (which is always the longest), Opposite (which doesn't touch the hypotenuse) and the adjacent (which is between the opposite and the hypotenuse).

Pythagoras is in the form of;

a² + b² = c²

We are given:

Diagonal = 27 inches

Length = 23.5 inches

Thus:

Height is:

h = √(27² - 23.5²)

h = 13.3 inches

Read more about Pythagoras Theorem at: https://brainly.com/question/654982

#SPJ1

A game requires players to roll a six-sided number cube and spin a spinner, like the one shown below. A circle shaped spinner divided in to 3 equal sections and named green ,blue, red. A needle is shown in center. Then, find the number of outcomes that represent rolling an odd number and landing on blue. What is the answer? 3 4 5 6? Please answer quick!!! I will get you Brainiest!!!

Answers

Using probability, there is only one outcome that represents rolling an odd number and landing on blue.

What exactly is probability?

Probability is a measure of the possibility or chance that an event will occur. It is represented by a number between 0 and 1, with 0 representing an improbable occurrence and 1 representing a certain event. A given event's probability is estimated by dividing the number of favourable outcomes by the total number of potential possibilities. Probability theory is frequently used to analyse and forecast the likelihood of occurrences in domains such as statistics, physics, economics, and finance.

Now,

The probability of rolling an odd number is 3/6, which can be simplified to 1/2. The probability of landing on blue= 1/3. Since the events of rolling an odd number and landing on blue are independent, we can multiply the probabilities to find the probability of both events occurring:

(1/2) × (1/3) = 1/6

So, there is only one outcome that represents rolling an odd number and landing on blue. The answer is 1.

To know more about probability visit the link

brainly.com/question/30034780

#SPJ1

What is the most upper (+3) or (-7)? Help please

Answers

Answer: Of the two numbers you provided, +3 is greater than -7. So, +3 is the most upper of the two numbers.

Answer: +3

Step-by-step explanation: Positive 3 is greater than negative 7. Therefore, +3 is the greater value.

Determine the scale ratio for this map given that the distance from Wellspring to Red Creek is 4 inches on the map and 10 miles in reality.

Answers

The scale ratio for this map is 0.0000063.

What is scale ratio?

Scale ratio is the ratio between the measurements of an object or distance on a map or drawing and the actual measurements of the object or distance it represents in real life. It allows us to compare and relate the sizes or distances of objects in a drawing or map to their actual sizes or distances in the real world.

The scale ratio can be found by dividing the distance on the map by the corresponding distance in reality:

scale ratio = distance on map / distance in reality

In this case, the distance from Wellspring to Red Creek is 4 inches on the map and 10 miles in reality. So the scale ratio is:

scale ratio = 4 inches / 10 miles

To simplify this ratio, we need to convert the units so that they are the same. Let's convert inches to miles:

1 mile = 63,360 inches

So, we can convert the inches on the map to miles as follows:

4 inches * (1 mile / 63,360 inches) = 0.000063 miles

Now we can rewrite the scale ratio as:

scale ratio = 0.000063 miles / 10 miles

Simplifying this ratio by canceling out the units of miles, we get:

scale ratio = 0.0000063

Therefore, the scale ratio for this map is 0.0000063.

To learn more about scale ratio visit the link:

https://brainly.com/question/30713474

#SPJ1

What is the smallest positive integer divisible by 6 and 2 you can write using at least one 2 and one 6?

Answers

The smallest positive integer divisible by 6 and 2 that can be written using at least one 2 and one 6 is 6.

The smallest positive integer that is divisible by both 2 and 6 is their least common multiple (LCM), which is equal to the product of the highest power of each prime factor that appears in the factorization of 2 and 6.

The prime factorization of 2 is simply 2, while the prime factorization of 6 is 2 × 3. The highest power of 2 that appears in the factorization of 6 is just 2 itself, so the LCM of 2 and 6 is 2 × 3 = 6.

We are asked to write this integer using at least one 2 and one 6. We can do this by simply writing 6, which is the LCM of 2 and 6 and is divisible by both of them. Since 6 contains one 2 and one 6, this meets the requirement of the problem. Therefore, the smallest positive integer divisible by 6 and 2 that can be written using at least one 2 and one 6 is 6.

Learn more about least common multiple here

brainly.com/question/30060162

#SPJ4

1. Suppose we have the following annual risk-free bonds Maturity Price Coupon Rate YTM 1 98 0% 2.01% 2 101 2.48% 3 103 2.91% 4 101 2% 1.73% 5 103 5% 4.32% 39 a) Find the zero rates for all 5 maturities Note: for an extra challenge, try using lincar algebra to find == A + where 98 00 -- 3 103 0 2 2 5 5 0 104 2 0 0 0 0 0 0 1020 5 105 5 1 b) Suppose we have a risk-free security which pays cash flows of $10 in one year, $25 in two years, and $100 in four years. Find its price

Answers

a) The zero rates for the five maturities are: 1 year is 2.01%, 2 years is 2.48%, 3 years is 2.77%, 4 years is 1.73%, and 5 years is 4.32%.

b) The price of the security is $128.31.

a) To find the zero rates for all 5 maturities, we can use the formula for the present value of a bond:

PV = C / [tex](1+r)^n[/tex]

where PV is the present value,

C is the coupon payment,

r is the zero rate, and

n is the number of years to maturity.

We can solve for r by rearranging the formula:

r = [tex](C/PV)^{(1/n) }[/tex]- 1

Using the bond data given in the question, we can calculate the zero rates for each maturity as follows:

For the 1-year bond, PV = 98 and C = 0, so r = 2.01%.

For the 2-year bond, PV = 101, C = 2.48, and n = 2, so r = 2.48%.

For the 3-year bond, PV = 103, C = 2.91, and n = 3, so r = 2.77%.

For the 4-year bond, PV = 101, C = 2, and n = 4, so r = 1.73%.

For the 5-year bond, PV = 103, C = 5, and n = 5, so r = 4.32%.

Alternatively, we can use linear algebra to find the zero rates. We can write the present value equation in matrix form:

PV = A × x

where A is a matrix of coefficients, x is a vector of unknowns (the zero rates), and PV is a vector of present values.

To solve for x, we can use the equation:

x = ([tex]A^{-1}[/tex]) x PV

where ([tex]A^{-1}[/tex]) is the inverse of matrix A.

Using this method, we can solve for the zero rates as follows:

[2.01% ]

[2.48% ]

[2.77% ] = x

[1.73% ]

[4.32% ]

PV = [tex]A^{-1}[/tex] x  [98]

                   [101]

                   [103]

                   [101]

                   [103]

PV =   [-0.0201]

          [ 0.0248]

          [ 0.0277]

          [-0.0173]

          [ 0.0432]

b) To find the price of the security which pays cash flows of $10 in one year, $25 in two years, and $100 in four years, we can use the formula for the present value of a series of cash flows:

PV = [tex]C1/(1+r)^1 + C2/(1+r)^2 + C3/(1+r)^4[/tex]

where PV is the present value, C1, C2, and C3 are the cash flows, r is the zero rate, and the exponents correspond to the number of years until each cash flow is received.

Using the zero rates calculated in part (a), we can calculate the present value of each cash flow:

PV1 = $10 /(1+2.01 % [tex])^1[/tex] = $9.80

PV2 = $25/(1+2.48%[tex])^2[/tex] = $22.15

PV3 = $100/(1+1.73%[tex])^4[/tex] = $81.36

Then, the price of the security is the sum of the present values:

PV = $9.80 + $22.15 + $81.36 = $128.31

Therefore, the price of the security is $128.31.

For similar question on price of the security

https://brainly.com/question/29245385

#SPJ11

Need answer by 11:45am
Question 8(Multiple Choice Worth 2 points)
(Creating Graphical Representations MC)

The number of milligrams of Vitamin C from 100 different gummy vitamins sold in the world was collected.

Which graphical representation would be most appropriate for the data, and why?

Box plot, because the median can easily be determined from the large set of data
Stem-and-leaf plot, because you can see the shape of the data
Histogram, because it shows each individual data point
Bar chart, because the data is categorical
Unlocked badge showing an astronaut’s boot touching down on the moon
See what the community says and unlock a badge

Answers

The graphical representation that is best and most appropriate for the data is Box plot, because the median can easily be determined from the large set of data. That is option A.

What is a box plot ?

The box plot is a type of graphical representation of data that gIves more than one detail about the data set such as;

minimum, first quartile, median, third quartile, and maximum.

Box plots allow you to compare multiple data sets better than others dues to the above listed features that it has.

Learn more about histogram here:

https://brainly.com/question/28164315

#SPJ1

The diameter of a circle is 12.5 cm. What is the circumference of the circle? Question 1 options: 19.63 cm 35. 25 cm 39.25 cm 78.5 cm please tell me quick!!!!

Answers

Answer:

The circumference of a circle is given by the formula:

C = πd

where C is the circumference, d is the diameter, and π is a mathematical constant approximately equal to 3.14.

In this case, the diameter of the circle is given as 12.5 cm.

Substituting this value into the formula, we get:

C = π(12.5)

C = 39.25 cm

Therefore, the circumference of the circle is 39.25 cm.

Hence, the answer is 39.25 cm.

Determine the equation of the hyperbola with and co-vertices (1, 5) and (-7, 5) and
asymptotes y = x+8 and y = -x +2.

Answers

The equation of the hyperbola is [tex](x + 3)^2/16 - (y - 5)^2/36[/tex] = 1.

What is hyperbola?

The collection of all points in a plane such that the distance between any point on the curve and two fixed points (referred to as the foci) is constant is known as a hyperbola. Hyperbolas are a sort of conic section. A hyperbola contains two distinct branches and has the appearance of two curving branches that are mirror reflections of one another. A hyperbola's center, vertices, co-vertices, foci, and asymptotes are some of its most important characteristics. The center, which is the point around which the hyperbola is symmetric, is the midway of the line segment connecting the vertices.

Given that, the co-vertices are (1, 5) and (-7, 5).

Now, using the midpoint formula we have:

center = ((1+(-7))/2, (5+5)/2) = (-3, 5)

Now, the distance between center and vertex is a = 4.

Also, the distance between the center and each co-vertex is b = 6.

Now, the equation of the hyperbola is:

[tex](x - (-3))^2/4^2 - (y - 5)^2/6^2 = 1\\(x + 3)^2/16 - (y - 5)^2/36 = 1[/tex]

Hence, the equation of the hyperbola is [tex](x + 3)^2/16 - (y - 5)^2/36[/tex] = 1.

Learn more about hyperbola here:

https://brainly.com/question/15697124

#SPJ1

.
The method of completing the square can be used to transform the equation x² - 6x + 8 = 0 into the form (x-p)² = q

Answers

Answer:

p = 3, q = 1

Step-by-step explanation:

well, let's think about what (x - p)² actually is.

let's do the multiplication (what a square is) :

(x - p)(x - p) = x² - px - px + p² = x² - 2px + p²

now, we compare the theoretical equation to the actual equation :

x² - 6x + 8 = 0

x² - 2px + p² = q

x² = x² check

-6x = -2px

-6 = -2p

p = -6/-2 = 3

that gives us

x² - 6x + 9 = q

compare this to

x² - 6x + 8 = 0

we subtract the second equation from the first

x² - 6x + 9 = q

- x² - 6x + 8 = 0

------------------------

0 0 1 = q

there you have it.

Solve the trigonometric equation for all values -5 sinπ/3x=0

Answers

The solutions to the equation sin(πx/3) = 0 in the interval -5 < x < 5 are x = -3, 0, and 3.

What is Trigonometric equation?

A trigonometric equation is an equation that involves trigonometric functions such as sine, cosine, tangent, or their inverses. Trigonometric equations arise in a variety of mathematical and scientific contexts, from solving geometric problems involving angles and triangles to modeling periodic phenomena in physics, engineering, and other fields.

Solving a trigonometric equation typically involves finding the values of the unknown variable that satisfy the equation within a certain interval. For example, the equation sin(x) = 0 has infinitely many solutions, but if we restrict the interval to [0, 2π], then the solutions are x = 0, π, 2π, which correspond to the x-intercepts of the sine function in that interval.

Here the equation sin(πx/3) = 0 has solutions whenever πx/3 is an integer multiple of π, since the sine function is zero at these values. Thus, we need to find all integers n such that πx/3 = nπ, or equivalently x = 3n for some integer n.

Since -5 < x < 5, we need to find all integers n such that -5 < 3n < 5. Dividing all sides by 3, we get -5/3 < n < 5/3. The only integers in this range are -1, 0, and 1. Therefore, the solutions to the equation sin(πx/3) = 0 in the interval -5 < x < 5 are x = -3, 0, and 3.

Learn more about trigonometric equation here,

https://brainly.com/question/24349828

#SPJ1

i need help with problem.

Answers

Answer:

[tex]y=5-2x[/tex]

Step-by-step explanation:

You have to give the equation in the form [tex]y=mx+c[/tex], where m is the gradient and c is the y-intercept (where the line crosses the y-axis).

From the graph, we can see the line crossed the y-axis at (0,5), so the y-intercept is (0,5), which means c is 5.

We can work out the gradient with the two points (2,1) and (1,3) by doing:

[tex]\frac{change in y}{change in x} =\frac{1-3}{2-1}=-2[/tex]
So the gradient of the line, m, is -2.

Thus the equation of the line is [tex]y=5-2x[/tex].

The answer to this question

Answers

The volume of the basketball is approximately 4.2x³.

How to solve for the volume

a. The formula for the volume of a sphere is:

V = (4/3)πr³

where r is the radius of the sphere. In this case, the radius is given as x, so we have:

V = (4/3)πx³ ≈ 4.2x³

Therefore, the volume of the basketball is approximately 4.2x³.

b. The volume of a cube is given by:

V = s³

where s is the length of one of its sides. Since the basketball touches all of the sides of the case, the length of one of its sides is equal to the diameter of the basketball plus the length of a side of the basketball. This is equal to 2x + 2x = 4x. Therefore, we have:

V = (4x)³ = 64x³

Therefore, the volume of the box is 64x³.

c. The volume of air in the box is equal to the volume of the box minus the volume of the basketball. We can substitute the formulas we obtained in parts (a) and (b) to get:

V_air = V_box - V_basketball = 64x³ - 4.2x³ = 59.8x³

Therefore, the volume of air in the box is approximately 59.8x³.

Read more on volume  here:https://brainly.com/question/27710307

#SPJ1

super sleep hotel has 1,700 guests who stayed for two nights and rented 150 rooms. how many guest nights did the hotel have during this period?

Answers

In linear equation, 4,000 guest nights did the hotel have during this period.

What is a linear equation in mathematics?

A linear equation is an algebraic equation of the form y=mx+b. m is the slope and b is the y-intercept. The above is sometimes called a "linear equation in two variables" where y and x are variables. 

According to question,

Guest nights = Number of guests × Number of nights

                       = 2,000 × 2

                       = 4,000

Learn more about linear equation

brainly.com/question/11897796

#SPJ1

Four friends want to play a game. In how many ways can the friends from teams, if both team dont have name

Answers

The friends can form teams in 6 different ways.

If the four friends want to form two teams, we can count the number of ways they can do this using combinations.

The number of ways to choose two people out of four is given by the combination formula,

C(4,2) = 4! / (2! * (4-2)!) = 6

We can list them as follows, where each team is represented by the letters A and B,

AB  |  CD

AC  |  BD

AD  |  BC

BC  |  AD

BD  |  AC

CD  |  AB

This means that there are 6 different ways the four friends can be split into two teams.

To learn more about teams here:

https://brainly.com/question/30622263

#SPJ4

An expression is shown.

3(-12.5)

What is the value of the expression?

Answers

Answer: -37.5

Step-by-step explanation: You can simply do this in the calculator by doing 3 times -12.5. the parenthesis is a sign to multiply

Answer:

the answer is -75/2= - 37.5

The range of which function is (2,00)?
O y = 2x
O y = 2(5*)
O y = 5x +2
O y = 5x + 2

Answers

B. y = 2(5^x) is an exponential function with a base of 5 and a coefficient of 2. As x varies, the output y will increase exponentially. The minimum value of this function is 2, and it approaches 0 as x approaches negative infinity. Therefore, the range of this function is (2, ∞).

So, the correct answer is B. y = 2(5^x)

The function which has range (2, ∞) is,

D) y = 5ˣ + 2

What is mean by Function?

A relation between a set of inputs having one output each is called a function. and an expression, rule, or law that defines a relationship between one variable (the independent variable) and another variable (the dependent variable).

Now, For a function: y = 5ˣ

Range is,

⇒ ( 0 , + ∞ )

And, we have:

y = 5ˣ + 2 ,

which is translated 2 units up.

So, the range is ( 2, + ∞ ).

Hence, The function which has range (2, ∞) is,

D ) y = 5ˣ + 2

Learn more about the function visit:

https://brainly.com/question/11624077

#SPJ5

. which one of the following statements is true? a. if you are given a sample percentage of 43%, you would need to know the sample size in order to convert this percentage to a proportion. b. the test statistic is affected by the size of the sample. c. the larger the p-value, the more evidence you have against the null hypothesis. d. we always begin a hypothesis test by assuming that the null hypothesis is false. e. none of the above statements are true.

Answers

If you are given a sample percentage of 43%, you would need to know the sample size in order to convert this percentage to a proportion.

The conversion formula is proportion = percentage/100. However, the proportion alone does not give information about the sample size, which is necessary for inference and hypothesis testing. The other statements are not true.

The test statistic is not affected by the sample size, but its value can be used to determine the significance of a hypothesis test. A larger p-value indicates weaker evidence against the null hypothesis, not stronger evidence. Finally, we assume the null hypothesis is true until we have sufficient evidence to reject it.

Learn more about percentage:

https://brainly.com/question/24304697

#SPJ11

n.2 multi-step word problems with positive rational numbers jvu you have prizes to reveal! go to your game board. on friday night, suzie babysat her cousin for 3 1 2 hours and earned $8.50 per hour. on saturday, she babysat for her neighbors for 4 1 2 hours. if she made a total of $72.50 from both babysitting jobs, how much did suzie earn per hour on saturday?

Answers

Answer:

  $9.50

Step-by-step explanation:

You want Suzie's hourly rate on Saturday if she babysat for 3.5 hours on Friday, earning 8.50 per hour, and for 4.5 hours on Saturday, earning a total of 72.50 from both jobs.

Earnings

For (hours, rates) of (h1, r1) and (h2, r2), Suzie's total earnings for the two jobs are ...

  earnings = h1·r1 +h2·r2

Filling in the known values, we can find r2:

  72.50 = 3.5·8.50 +4.5·r2

  72.50 = 29.75 +4.5·r2 . . . . . . . simplify

  42.75 = 4.5·r2 . . . . . . . . . . . subtract 29.75

  9.50 = r2 . . . . . . . . . . . . divide by 4.5

Suzie earned $9.50 per hour on Saturday.

__

Additional comment

The steps of the "multistep" problem are ...

find Friday's earningssubtract that from the total to find Saturday's earningsdivide by Saturday's hours to find the hourly rate

Effectively, these are the steps to solving the equation we wrote.

true or false: we conduct a test of hypothesis by assuming that ha is correct, since that is the hypothesis we are trying to show is true. group of answer choices true false

Answers

Given statement "We conduct a test of hypothesis by assuming that ha is correct, since that is the hypothesis we are trying to show is true." is true. Because we start by assuming that the null hypothesis is correct, not the alternative hypothesis, when conducting a test of the hypothesis.

When conducting a test of hypothesis, we do not assume that Ha (the alternative hypothesis) is correct.

Instead, we begin by assuming that the null hypothesis (H0) is true.

The null hypothesis typically represents a statement of "no effect" or "no difference" between two groups or variables.

The alternative hypothesis (Ha) represents the statement we are trying to prove or gather evidence for but we must start with the assumption that the null hypothesis is correct.

State the null hypothesis (H0) and the alternative hypothesis (Ha).

The null hypothesis typically represents the status quo or a baseline assumption, while the alternative hypothesis represents the claim you want to prove or gather evidence for.

Determine the level of significance (α), which is the probability of rejecting the null hypothesis when it is actually true. Common significance levels are 0.05, 0.01, and 0.001.

Select an appropriate test statistic, which depends on the type of data and the hypothesis being tested. Examples include the t-test, chi-square test, and ANOVA.

Collect data and calculate the value of the test statistic based on the data.

Compare the calculated test statistic value to the critical value for the chosen level of significance.

If the test statistic is more extreme than the critical value, we reject the null hypothesis in favor of the alternative hypothesis.

Otherwise, we fail to reject the null hypothesis.

Hence, the statement is false.

For similar question on hypothesis.

https://brainly.com/question/27170372

#SPJ11

Evaluate the following.
Write an exponential function of the form y = ab^x that has the given points
(−1,6 3/4), (2, 1-4)

Answers

Answer:

Step-by-step explanation:

y = abx

a is the y-intercept

y = 16bx

Now substitute 2 for x and 1296 for y

1296 = 16(b)2

81 = b2

b = 9

y = 16(9)x

Please help me with this homework

Answers

Answer:sasa

Step-by-step explanation:

what is the probability that the largest among these random samples is greater than the population median?

Answers

The probability that the largest of n random samples is greater than the population median M is bounded above by[tex]1 - F(M)^(n-1) \times F(X(n))[/tex].

Assumptions about the population and the sampling method.

Let's assume that the population has a continuous probability distribution with a well-defined median, and that we are taking independent random samples from this population.

Let [tex]X1, X2, ..., Xn[/tex] be the random samples that we take from the population, where n is the sample size.

Let M be the population median.

The probability that the largest of these random samples, denoted by X(n), is greater than M.

Cumulative distribution function (CDF) of the population distribution to calculate this probability.

The CDF gives the probability that a random variable takes on a value less than or equal to a given number.

Let F(x) be the CDF of the population distribution.

Then, the probability that X(n) is greater than M is:

[tex]P(X(n) > M) = 1 - P(X(n) < = M)[/tex]

Since we are assuming that the samples are independent, the joint probability of the samples is the product of their individual probabilities:

[tex]P(X1 < = x1, X2 < = x2, ..., Xn < = xn) = P(X1 < = x1) \times P(X2 < = x2) \times ... \times P(Xn < = xn)[/tex]

For any x <= M, we have:

[tex]P(Xi < = x) < = P(Xi < = M) for i = 1, 2, ..., n[/tex]

Therefore,

[tex]P(X1 < = x, X2 < = x, ..., Xn < = x) < = P(X1 < = M, X2 < = M, ..., Xn < = M) = F(M)^n[/tex]

Using the complement rule and the fact that the samples are identically distributed, we get:

[tex]P(X(n) > M) = 1 - P(X(n) < = M)[/tex]

= [tex]1 - P(X1 < = M, X2 < = M, ..., X(n) < = M)[/tex]

=[tex]1 - [P(X1 < = M) \times P(X2 < = M) \times ... \times P(X(n-1) < = M) \times P(X(n) < = M)][/tex]

[tex]< = 1 - F(M)^(n-1) \times F(X(n))[/tex]

Probability depends on the sample size n and the distribution of the population.

If the population is symmetric around its median, the probability is 0.5 for any sample size.

As the sample size increases, the probability generally increases, but the rate of increase depends on the population distribution.

For similar questions on Median

https://brainly.com/question/26177250

#SPJ11

Hugo made a bunch of batches of pancakes for his summer camp. For each batch, Hugo used 1/3 cup of milk. Hugo used a total of 3 cups of milk. Let b represent the number of batches Hugo mad

Answers

The number of batches of pancakes made by Hugo using 3 cups of milk is equal to 9 batches.

Number of cups of milk used by Hugo for each batch =  1/3 cup of milk ,

The total amount of milk used for b batches would be,

Total milk = (1/3) × b

The total milk used was 3 cups,

Substitute the value in the equation we have,

⇒ (1/3) × b = 3

Solve for b we get,

Multiply both sides of the equation by the reciprocal of 1/3,

Reciprocal of ( 1/3 ) = 3/1 or simply 3

⇒ (1/3) × b × 3 = 3 × 3

⇒ b = 9

Therefore, Hugo made 9 batches of pancakes.

Learn more about batches here

brainly.com/question/856537

#SPJ4

The above question is incomplete, the complete question is:

Hugo made a bunch of batches of pancakes for his summer camp. For each batch, Hugo used 1/3 cup of milk. Hugo used a total of 3 cups of milk. Let b represent the number of batches Hugo made . Find the value of b?

Maureen bought 3/4 of a pound of small binder clips and 3/4 of a pound of jumbo binderclips. How much did she spend

Answers

If Alice has $50 and spends 4/5 of her money, she spends $40.

A fraction is a mathematical expression that represents a part of a whole or a ratio between two quantities. Fractions are commonly represented using two numbers separated by a horizontal line, where the top number is called the numerator and the bottom number is called the denominator.

Alice spends 4/5 of her money, which is equivalent to 0.8 of her money.

To find out how much Alice spends, we can multiply her total money ($50) by 0.8

= $50 x 0.8

Multiply the numbers

= $40

Therefore, Alice spends $40.

Learn more about fraction here

brainly.com/question/8482939

#SPJ4

I have solved the question in general, as the given question is incomplete.

The complete question is:

Alice has $50 and spends 4/5 of her money. How much does Alice spend?

sue works 5 out of the 7 days of the week. how many possible schedules are there to work on tuesday or friday or both?

Answers

Sue works 5 out of 7 days a week, which implies that she has two days off. We need to discover how numerous conceivable plans there are for her to work on Tuesday or Friday or both.

There are two cases to consider:

1. Sue works on Tuesday as it were, Friday as it were, or both Tuesday and Friday.

2. Sue does not work on Tuesday or Friday.

For the primary case, there are three conceivable outcomes:

1. Sue works on Tuesday as it were and has Friday off.

2. Sue works on Friday as it were and has Tuesday off.

3. Sue works on both Tuesdays and Fridays.

For the moment case, there are two conceivable outcomes:

1. Sue works on one of the other 5 days of the week and has both Tuesday and Friday off.

2. Sue has Tuesday and Friday off.

In this manner, there are added up to 3 + 2 = 5 conceivable plans for Sue to work on Tuesday or Friday or both. 

To know more about possible schedules for the week refer to this :

https://brainly.com/question/23689163

#SPJ4

an airline passenger is planning a trip that involves three connecting flights that leave from airports a, b, and c, respectively. the first flight leaves airport a every hour, beginning at 8:00 a.m., and arrives at airport b 2 1/2 hours later. the second flight leaves airport b every 20 minutes, beginning at 8: 00 a.m., and arrives at airport c 1 1/6hours later. the third flight leaves airport c every hour, beginning at 8:45 a.m. what is the least total amount of time the passenger must spend between flights if all flights keep to their schedules?

Answers

An airline passenger is planning a trip with three connecting flights from airports A, B, and C.

The least total amount of time the passenger must spend between flights, assuming all flights keep to their schedules, is 55 minutes.

This occurs when the passenger takes the first flight from airport A at 8:00 a.m., arriving at airport B at 10:30 a.m., catches the second flight from airport B at 10:40 a.m., arriving at airport C at 11:50 a.m., and then takes the third flight from airport C at 12:45 p.m.

Learn more about airline:

https://brainly.com/question/29579489

#SPJ11

In the month of January, Sasha had a balance of $3200 on her credit card. She made a payment of $300 and left the remaining balance to be paid later. How much interest will she pay this month if her APR is 18. 75%? Round to the nearest cent. Question 3 options:

$35. 10


$543. 75


$46. 19


$4. 50

Answers

The total amount of interest she will have to pay for this month considering her APR is 18.75% is $362.5. therefore the correct option is Option D

In order to calculate the credit card interest we have to divide the annual percentage rate by 12.

the formula for evaluating the interest on the given credit card is

Interest = ( balance x APR)/12

here,

balance = outstanding balance

APR = annual percentage rate

Given,

Sasha had a balance of $3200 from which a purchase of $300 was made the remaining balance is to be paid later.

then, Sasha's

outstanding balance = $2900

APR = 18.75%

therefore, Sasha's monthly interest is

18.75/12 => 1.5625%

now, calculating the interest on the given credit card

Interest = (2900 x 1.5625) /12

Interest = $362.5

The total amount of interest she will have to pay for this month considering her APR is 18.75% is $362.5

To learn more about interest,

https://brainly.com/question/20690803

#SPJ4

The complete question is

In the month of January, Sasha had a balance of $3200 on her credit card. She made a payment of $300 and left the remaining balance to be paid later. How much interest will she pay this month if her APR is 18.75%? Round to the nearest cent.

a) $35. 10

b) $543. 75

c) $46. 19

d) $362.5

A primary credit cardholder's card has an APR of 22. 99%. The current monthly balance, before interest, is $4,528. 34. Determine how much more the cardholder will pay, making monthly payments of $200, until the balance is paid off, instead of paying off the current balance in full

Answers

The cardholder will pay an additional $1,471.66 in interest by making monthly payments of $200 until the balance is paid off instead of paying off the current balance in full.

First, we need to calculate the total interest that will accrue on the current balance of $4,528.34. We can do this using the formula

Interest = Balance x (APR/12)

where APR is the annual percentage rate and is divided by 12 to get the monthly interest rate. Plugging in the values, we get:

credit card Interest = $4,528.34 x (22.99%/12) = $87.80

So the total interest that will accrue on the current balance is $87.80.

Next, we need to calculate how long it will take to pay off the balance by making monthly payments of $200. We can use a credit card repayment calculator to do this, but we'll use a simplified formula here

Months = -log(1 - (Balance x (APR/12))/Payment) / log(1 + (APR/12))

where Payment is the monthly payment amount. Plugging in the values, we get

Months = -log(1 - ($4,528.34 x (22.99%/12))/$200) / log(1 + (22.99%/12)) = 29.6 months

So it will take about 30 months (or 2.5 years) to pay off the balance by making monthly payments of $200.

Finally, we can calculate how much more the cardholder will pay in total by subtracting the current balance from the total amount paid over 30 months

Total amount paid = $200 x 30 = $6,000

Total interest paid = $6,000 - $4,528.34 = $1,471.66

Learn more about Credit card interest here

brainly.com/question/29641204

#SPJ4

What is the solution to the system of equations graphed below? A. (0,6) B. (6,0) C. (0,3) D. (1,5)

Answers

The correct option is - D. (1,5). The solution to the system of equations for the given graph is at (1,5).

Explain about the solution of system of equations:

A collection of values for a variable that simultaneously fulfil each equation is the solution to a system of equations. A system of equations must be solved by identifying all possible sets of variable values that make up the system's solutions.

The points where the lines representing the intersections where two linear equations intersect are referred to as the conclusion of a linear equation. In other words, the set of all feasible values for the variables that satisfy the specified linear equation constitutes the solution set of both the system of linear equations.

For the given graph, the solution of the system of equation is obtained as the point where the lines intersect.

The two lines in the graph intersect at the (1,5). Thus, the solution to the system of equations for the given graph is at (1,5).

Know more about the system of equations

https://brainly.com/question/25976025

#SPJ1

Other Questions
a property sold for $250,000. the reproduction cost of the building was $380,000 and it was 60 epreciated. by extraction, what is the value of the land? identify and describe the characteristic properties of five common acids used in industry. give some examples of the typical uses of each. How was your image as a reader shaped by your environment when you grew up as a young child? Reflect on the following: Types of literacies that you were exposed to as a young child. Opportunities that were created to engage with literacy and role adults have played to shape your understanding of literacy. which theory for the settlement of the new world relies on a passage using canoes, rafts, or other forms of water transport? 4. if 1 drop of acid is equal to 50 microliter. calculate the concentration of h ion and the ph of the solution when 1 drop of 0.25 m hcl is added to 3 ml water. does that conform to your observation in part d. if not, why? the equilibrium concentrations for fe3 and scn- are 5.0 x 10-4 m and 7.5 x 10-4 m. what is the equilibrium concentration for fe(scn)2 ? when providing education to a client diagnosed with a benign bone tumor, the nurse should emphasize the fact that benign tumors primarily: the odd skipped gene in drosophila received its name because mutations in that gene often lead to embryos missing the odd-numbered segments. based on this information, what is the most likely class of gene to which odd skipped would belong? I need help with this please Describe what is unique about the island of Iceland describe the strengths and weakness of the major types of advertising media and give an example of products that would be advertised using each of them. jimmy is screaming at people out the car window, trying to push into other lanes and snarling at the passenger in his car. what is the best way to describe his personality? Question 7:- Explain the relationship between the discount (interest) rate and the Present Value (PV) of any future cash flows. Question 8: Explain the relationship between the discount (interest) rate and the Future Value (FV) of any future cash flows. Which parts are the female reproductive parts of the flower? which type of document addresses the specific concerns realted to access given to administrators and certain support staff? True/False: Paraffin wax (d = 0.910 g/cm3) would float in ethanol (d = 789 kg/L) what technique allows researchers to examine the relationship between two or more variables, while systematically controlling for one or more potential third variables? NMENTSCOURSESWRITERAssignment-13 Quiz 2Attempt 3 of 3HSECTION 1 OF 1In at least one hundred words, discuss the significance of the conversation that occurs in "Hills Like White ElephantsHow do Jig and the American relate to one another? How do they understand one another?QUUP I did not get the answer Deposits of 70 are placed into a fund at the end of each year for 10 years. The effective annual interest rate is 8%. Calculate the accumulated value of the series of payments at the end of the 10th yeara.1,014.06 b.770.69 c.932.93 d.1.095.18 e.1851.81 self-employed persons can make contributions for their retirement into a special tax-deferred account called a keogh account. suppose you are able to contribute $20,000 into this account at the end of each year. how much will you have at the end of 20 years if the account pays 3% annual interest? (round your answer to the nearest cent.)